simplify: -6(2x-9)

any help would be appreciated!​

Answers

Answer 1

Answer:

-12x+54

Step-by-step explanation:

Answer 2

[tex]\huge\text{Hey there!}[/tex]

[tex]\huge\text{In order for you to simplify, you need to}[/tex] [tex]\huge\text{ distribute}[/tex]

[tex]\huge\text{The formula is: a(b + c) = a(b) + a(c) = ab + ac}[/tex]

[tex]\huge\text{Simplify: -6(2x - 9)}[/tex]

[tex]\huge\text{-6(2x) = -12x}\\\huge\text{-6(-9) = 54}[/tex]

[tex]\boxed{\boxed{\huge\text{Answer: -12x + 54}}}\huge\checkmark[/tex]

[tex]\text{Good luck on your assignment and enjoy your day!}[/tex]

~[tex]\frak{LoveYourselfFirst:)}[/tex]


Related Questions

Which expression represents the number rewritten in a+ bi form?

Answers

[tex]\bf \textit{recalling that }~\hfill i^4=1~\hfill i^3=-i~\hfill i^2=-1 \\\\[-0.35em] \rule{34em}{0.25pt}\\\\ 5i^4+2i^3+8i^2+\sqrt{-4}\implies 5(1)+2(-i)+8(-1)+\sqrt{-1\cdot 4} \\\\\\ 5-2i-8+\sqrt{-1}\cdot \sqrt{2^2}\implies -2i-3+i\cdot 2\implies ~~\begin{matrix} -2i \\[-0.7em]\cline{1-1}\\[-5pt]\end{matrix}~~-3~~\begin{matrix} +2i \\[-0.7em]\cline{1-1}\\[-5pt]\end{matrix} \\\\[-0.35em] \rule{34em}{0.25pt}\\\\ ~\hfill -3+0i~\hfill[/tex]

b is the midpoint of AC and E is the midpoint of BD if A(-9,-4) C(-1,6) and E(-4,-3) find the coirdinates of D

Answers

Answer:

Step-by-step explanation:

[tex]B=(\frac{-9-1}{2},\frac{-4+6}{2} ) (midpoint formula)\\B=(-5,1)\\\\Then using midpoint formula again,\\E=(-4,3)=(\frac{-5+x}{2} =-4,\frac{1+y}{2} =3)\\so \\x=-3\\y=5\\\\D=(-3,5)[/tex]:

Find the distance across the lake. Assume the triangles are similar

Answers

Check the picture below.

The distance across the lake is 210 meters, and this is found using the concept of similar triangles and proportionality.

The correct answer is option B.

To find the distance across the lake, we can use the concept of similar triangles. Given that the triangles are similar, we can set up a proportion to find the missing side length.

Let's label the sides of the larger triangle as A, B, and C and the sides of the smaller triangle as X, Y, and Z. The given values are:

A = 70 m (the length of the larger triangle)

X = 20 m (the length of the smaller triangle)

Y = 60 m (the width of the smaller triangle)

We want to find side C, which represents the distance across the lake.

We can set up a proportion between the corresponding sides of the two similar triangles:

A / X = B / Y = C / Z

Plugging in the given values:

70 m / 20 m = C / 60 m

Now, solve for C by cross-multiplying:

70 m * 60 m = 20 m * C

4200 m² = 20 m * C

To isolate C, divide both sides by 20 m:

C = 4200 m² / 20 m

C = 210 m

So, the distance across the lake (side C) is 210 meters. This is the solution based on the concept of similar triangles and the given side lengths.

Therefore, from the given options the correct one is B.

For more such information on: distance

https://brainly.com/question/26550516

#SPJ2

If you apply the changes below to the absolute value parent function, f(x) = |x|,what is the equation of the new function?
• Shift 4 units to the right.
• Shift 6 units up.

Answers

Answer:

Option B [tex]g\left(x\right)=\left|x-4\right|+6[/tex]

Step-by-step explanation:

we have

[tex]f\left(x\right)=\left|x\right|[/tex]

The vertex is the point (0,0)

If to the parent function apply

Shift 4 units to the right and Shift 6 units up

The rule of the translation is

(x,y) ------> (x+4,y+6)

so

The vertex of the new function is (4,6)

therefore

the equation of the new function is

[tex]g\left(x\right)=\left|x-4\right|+6[/tex]

Answer:

just entered the answer and it was |x-4|+6

hope that helps!

Step-by-step explanation:

What is the prime factorization of 31?
Enter your answer as a product of prime numbers, like 2 x 3, or as a single prime number, like 17.​

Answers

Answer:

Step-by-step explanation:

31 is prime. It can't be factored. The way I'm reading the directions, you should enter 31.

Answer:

31

Step-by-step explanation:

The prime factorization of 31 is 1 x 31 = 31.

You have to multiply the number by 1 to find its prime factorization.

However, it is usually written as just 31.

bob spent 3/8 of his birthday money at a baseball game and 5/12 ona new bat and glove.what fraction of his birthday money did bob spend?

Answers

Answer:

19/24

Step-by-step explanation:

Since you are looking for the fraction he spend, you have to add 3/8 and 5/12 with one another. To do this you have to change those fractions to have the same denominators. so you multiply 3/8 by 3/3 to get 9/24 and you multiply 5/12 by 2/2 to get 10/24. You then add 9/24 with 10/24 to get 19/24. Since you cannot simplify this further, 19/24 is your answer.

Bob spent [tex]\frac{19}{24}[/tex] of his birthday money

What is fraction?"It is a number is expressed as a quotient, in which the numerator is divided by the denominator.""It is used to represent the part of the whole thing. "

For given question,

Bob spent 3/8 of his birthday money at a baseball game and 5/12 on a new bat and glove.

We need to find the fraction of his birthday money he spent.

so we add given two fractions.

[tex]\frac{3}{8}+ \frac{5}{12}\\\\ =\frac{3\times 3}{8\times 3}+ \frac{5\times 2}{12\times 2}\\\\=\frac{9}{24}+ \frac{10}{24}\\\\=\frac{9+10}{24}\\\\ =\frac{19}{24}[/tex]

Therefore, Bob spent [tex]\frac{19}{24}[/tex] of his birthday money.

Learn more about the fraction here:

https://brainly.com/question/12151403

#SPJ2

Number 10 please I don’t know how to do it and if you could also do 11

Answers

Answer:

C) -3

Step-by-step explanation:

x = 1, y = 12

x = 2, y = 9

x = 3, y = 6

x = 4, y = 3

The formula of a slope:

[tex]m=\dfrac{y_2-y_1}{x_2-x_1}[/tex]

[tex]m=\dfrac{9-12}{2-1}=\dfrac{-3}{1}=-3[/tex]

Which graph best represents the solution to the following pair of equations? y = 2x − 10 y = −x + 18

Answers

Answer:(6,12)

Step-by-step explanation:

Sin x + cos x = cos x/1-tanx + sin x/1-cot x. Verify the identity. Explain each step please!

Answers

Answer:

[tex]sinx+cosx=\frac{cosx}{1-tanx}+\frac{sinx}{1-cotx}\\[/tex] proved.

Step-by-step explanation:

[tex]sinx+cosx=\frac{cosx}{1-tanx}+\frac{sinx}{1-cotx}\\[/tex]

Taking R.H.S

[tex]\frac{cosx}{1-tanx}+\frac{sinx}{1-cotx}\\[/tex]

Multiply and divide first term by cos x and second term by sinx

[tex]=\frac{cosx*cosx}{cosx(1-tanx)}+\frac{sinx*sinx}{sinx(1-cotx)}[/tex]

we know tanx = sinx/cosx and cotx = cosx/sinx

[tex]=\frac{cos^2x}{cosx(1-\frac{sinx}{cosx} )}+\frac{sin^2x}{sinx(1-\frac{cosx}{sinx})}\\=\frac{cos^2x}{cosx-sinx}+\frac{sin^2x}{sinx-cosx}[/tex]

Taking minus(-) sign common from second term

[tex]=\frac{cos^2x}{cosx-sinx}-\frac{sin^2x}{cosx-sinx}[/tex]

taking LCM of cosx-sinx and cosx-sinx is cosx-sinx

[tex]=\frac{cos^2x-sin^2x}{cosx-sinx}[/tex]

We know a^2-b^2 = (a+b)(a-b), Applying this formula:

[tex]=\frac{(cosx+sinx)(cosx-sinx)}{cosx-sinx}\\=cosx+sinx\\=L.H.S[/tex]

Hence proved

misty's surgery lasted 2 1 hr. convert time to seconds?

Answers

= 21.0000 hours

= 1260.00 minutes

= 75600 seconds

For this case we must convert 21 hours to seconds!

We have that by definition, 1 hour equals 3600 seconds. Then, making a rule of three we have:

1h ------------> 3600s

21h ----------> x

Where "x" represents the number of seconds equivalent to 21 hours.

[tex]x = \frac {21 * 3600} {1} = 75600[/tex]

Thus, 21 hours represent 75600 seconds.

Answer:

75600 seconds.

The measure of central angle XYZ is 3pi/4 radians. What is the area of the shaded sector?
32pi units2
85 ..
96 ..
256 ..

Answers

Answer:

96π units²

Step-by-step explanation:

area of shaded sector (A) = area of circle × fraction of circle

A = πr² × [tex]\frac{\frac{3\pi }{4} }{2\pi }[/tex]

   = 16² × [tex]\frac{3\pi }{8}[/tex]

   = 256 × [tex]\frac{3\pi }{8}[/tex]

   = 32 × 3π

   = 96π units²

Answer:

Area of sector : 96 π unit².

Step-by-step explanation:

Given : The measure of central angle XYZ is 3pi/4 radians.

To find : What is the area of the shaded sector?

Solution: We have given central angle XYZ is 3pi/4 radians.

Area of sector : [tex]\frac{1}{2}[/tex] (radius)²* central angle.

Plug the values central angle  = [tex]\frac{3\pi }{4}[/tex] , radius = 16 units.

Then ,

Area of sector : [tex]\frac{1}{2}[/tex] (16)²* [tex]\frac{3\pi }{4}[/tex].

Area of sector : [tex]\frac{1}{2}[/tex] * 256 * [tex]\frac{3\pi }{4}[/tex].

Area of sector :  128 * [tex]\frac{3\pi }{4}[/tex].

Area of sector : 32 * 3 π

Area of sector : 96 π unit².

Therefore, Area of sector : 96 π unit².

what are the x intercepts, vertex and axis of symmetry of y=-(x+1)(x-5)

Answers

Answer:

x intercepts: (-1, 0) and (5, 0)

vertex: (2, 9)

axis of symmetry: x = 2

Step-by-step explanation:

The x intercepts are where y = 0.

0 = -(x + 1)(x − 5)

x = -1, x = 5

So the x intercepts are (-1, 0) and (5, 0).

The x coordinate of the vertex is halfway between the x intercepts.

x = (-1 + 5) / 2

x = 2

So the vertex is at (2, 9).

The axis of symmetry passes through the vertex.

x = 2

The cosine of 23° is equivalent to the sine of what angle

Answers

Answer:

So 67 degrees is one value that we can take the sine of such that is equal to cos(23 degrees).

(There are more values since we can go around the circle from 67 degrees numerous times.)

Step-by-step explanation:

You can use a co-function identity.

The co-function of sine is cosine just like the co-function of cosine is sine.

Notice that cosine is co-(sine).

Anyways co-functions have this identity:

[tex]\cos(90^\circ-x)=\sin(x)[/tex]

or

[tex]\sin(90^\circ-x)=\cos(x)[/tex]

You can prove those drawing a right triangle.

I drew a triangle in my picture just so I can have something to reference proving both of the identities I just wrote:

The sum of the angles is 180.

So 90+x+(missing angle)=180.

Let's solve for the missing angle.

Subtract 90 on both sides:

x+(missing angle)=90

Subtract x on both sides:

(missing angle)=90-x.

So the missing angle has measurement (90-x).

So cos(90-x)=a/c

and sin(x)=a/c.

Since cos(90-x) and sin(x) have the same value of a/c, then one can conclude that cos(90-x)=sin(x).

We can do this also for cos(x) and sin(90-x).

cos(x)=b/c

sin(90-x)=b/c

This means sin(90-x)=cos(x).

So back to the problem:

cos(23)=sin(90-23)

cos(23)=sin(67)

So 67 degrees is one value that we can take the sine of such that is equal to cos(23 degrees).

A wholesaler receives an order for 8000 pounds of produce. If the cost of shipping is $90 per ton, how much will the wholesaler have to pay for this shipment?

Answers

Answer:360

Step-by-step explanation:

1 ton = 2000 pounds

8000/2000 =4

4x90= 360.00

Answer is 360.00 for shipping

[tex]\huge{\boxed{\$360}}[/tex]

Each pound contains [tex]2000[/tex] pounds, so divide [tex]8000 \div 2000[/tex] to find the number of tons. [tex]8000 \div 2000=4[/tex]

Now, just multiply [tex]\$90*4=\boxed{\$360}[/tex]

I NEED HELP NOW


The graph shown is a scatter plot:



Which point on the scatter plot is an outlier?

Point A
Point B
Point C
Point D

Answers

Answer: The outlier in this scatter plot is Point D.

Step-by-step explanation:

The reason its Point D because its the only one that is the farest away from the rest of the points on the plot.

Hello there!Answer: Point D

The reason why "Point D" would be the correct answer because the point is no where near the "trend" of the scatter plot.

When you look at the scatter plot, you would notice that there following a "trend" when the graph increases.

However, you would notice that point D is not really near there, therefore making it an outlier.

An outlier is pretty much something that is left out, and in this case, point D would be the outlier since it's left out of the typical growth of the scatter plot.

I hope this helps!Best regards,MasterInvestor

Examine the following system of inequalities.
{y <−1/4x+4 and y>(x+4)^2
Which option shows the graph of the system?

Dotted linear inequality shaded above passes through (0, 4) & (4,5). Dotted parabolic inequality shaded below passes through (negative 6,4), (negative 4, 0) & (negative 2, 4).

Dotted linear inequality shaded below passes through (0, 4) & (4,3). Dotted parabolic inequality shaded above passes through points (negative 6,4), (negative 4, 0) & (negative 2, 4).

Dotted linear inequality shaded below passes through (0, 4) & (4,5). Dotted parabolic inequality shaded above passes through points (negative 6,4), (negative 4, 0) & (negative 2, 4).

Dotted linear inequality shaded above passes through (0, 4) & (4,3). Dotted parabolic inequality shaded below passes through (negative 6,4), (negative 4, 0) & (negative 2, 4).


Answers

Answer:

Dotted linear inequality shaded below passes through (0, 4) & (4,3). Dotted parabolic inequality shaded above passes through points (negative 6,4), (negative 4, 0) & (negative 2, 4).

Step-by-step explanation:

Hello! Let me help you to find the correct option to this problem. First of all, we have the following system of inequalities:

[tex]\left\{ \begin{array}{c}y< -\frac{1}{4}x+4\\y>(x+4)^{2}\end{array}\right.[/tex]

To solve this, let's write the following equations:

FIRST:

[tex]y=-\frac{1}{4}x+4[/tex]

This is a linear function written in slope-intercept form as [tex]y=mx+b[/tex]. So, the slope [tex]m=-\frac{1}{4}[/tex] and the y-intercept is [tex]b=4[/tex]. Since in the inequality we have the symbol < then the graph of the line must be dotted. To get the shaded region, let's take a point, say, [tex](0, 0)[/tex] and let's test whether the region is above or below the graph. So:

[tex]y< -\frac{1}{4}x+4 \\ \\ Let \ x=y=0 \\ \\ 0<-\frac{1}{4}(0)+4 \\ \\ 0<4 \ True![/tex]

Since the expression is true, then the region is the one including point [tex](0, 0)[/tex], that is, it's shaded below.

SECOND:

[tex]y=(x+4)^{2}[/tex]

This is a parabola that opens upward and whose vertex is [tex](-4,0)[/tex]. Since in the inequality we have the symbol > then the graph of the parabola must be dotted. Let's take the same point [tex](0, 0)[/tex] to test whether the region is above or below the graph. So:

[tex]y>(x+4)^{2} \\ \\ Let \ x=y=0 \\ \\ 0>(0+4)^2\\ \\ 0>16 \ False![/tex]

Since the expression is false, then the region is the one that doesn't include point [tex](0, 0)[/tex], that is, it's shaded above

____________________

On the other hand, testing points (0, 4)  and (4,3) on the linear function:

[tex]y=-\frac{1}{4}x+4 \\ \\ \\ \bullet \ (0,4): \\ \\ y=-\frac{1}{4}(0)+4 \therefore y=4 \\ \\ \\ \bullet \ (4,3): \\ \\ y=-\frac{1}{4}(4)+4 \therefore y=3[/tex]

So the line passes through these two points.

Now, testing points (negative 6,4), (negative 4, 0) & (negative 2, 4) on the parabola:

[tex]y=(x+4)^2 \\ \\ \\ \bullet \ (-6,4): \\ \\ y=(-6+4)^2 \therefore y=(-2)^2 \therefore y=4 \\ \\ \\ \bullet \ (-4,0): \\ \\ y=(-4+4)^2 \therefore y=0 \\ \\ \\ \bullet \ (-2,4): \\ \\ y=(-2+4)^2 \therefore y=(2)^2 \therefore y=4[/tex]

So the line passes through these three points.

Finally, the shaded region is shown below.

Write an equation in slope intercept from of the line that passes through the point (3,-2) with slope -2

Answers

[tex]\bf (\stackrel{x_1}{3}~,~\stackrel{y_1}{-2})~\hspace{10em} slope = m\implies -2 \\\\\\ \begin{array}{|c|ll} \cline{1-1} \textit{point-slope form}\\ \cline{1-1} \\ y-y_1=m(x-x_1) \\\\ \cline{1-1} \end{array}\implies y-(-2)=-2(x-3) \\\\\\ y+2=-2x+6\implies y=-2x+4[/tex]

HELP ME WITH THIS QUESTION ✔
THANKS YOU !!

Answers

[tex]\text{Hey there!}[/tex]

[tex]\huge\text{Decimals, fractions, \& negatives are below 0!}[/tex]

[tex]2\dfrac{21}{30}\ = 2\times30+21 \ = 2\times 30 = 60+21= 81\rightarrow \dfrac{81}{30}[/tex]

[tex]2\dfrac{1}{2}\ = 2\times2+1=2\times2=4+1=5\rightarrow\dfrac{5}{2}[/tex]

[tex]-\dfrac{32}{40}=-\dfrac{32}{40}[/tex]

[tex]\boxed{\boxed{\huge\text{Answer:}-\dfrac{32}{40}, \ 2\dfrac{1}{2}, \ 2\dfrac{21}{30}}}\huge\checkmark[/tex]

[tex]\text{Good luck on your assignment and enjoy your day!}[/tex]

~[tex]\frak{LoveYourselfFirst:)}[/tex]

What is the point slope form of a line that has a slope of 3 and passes through the point (-1,4)

Answers

Answer:

y - 4 = 3(x- (-1)     --->      y-4 = 3(x+1)

Step-by-step explanation:

y - 4 = 3(x + 1)

y - 4 = 3x + 3

y = 3x + 7

then plug in (-1,4)

4 = 3(-1) + 7

4 = -3 + 7

4 = 4

Point slope form looks like:

y - y₁ = m(x - x₁)

Answer:

Y-4=3[(x-(-1)]

Step-by-step explanation:

This answer is corect i got it right on my questions

Which proportion could be used to find the length of side b?​

Answers

Answer:

B

Step-by-step explanation:

Using the Sine Rule in ΔABC

[tex]\frac{a}{sinA}[/tex] = [tex]\frac{b}{sinB}[/tex] = [tex]\frac{c}{sinC}[/tex]

∠C = 180° - (82 + 58)° = 180° - 140° = 40°

Completing values in the above formula gives

[tex]\frac{a}{sin58}[/tex] = [tex]\frac{b}{sin82}[/tex] = [tex]\frac{8.4}{sin40}[/tex]

We require a pair of ratios which contain b and 3 known quantities, that is

[tex]\frac{b}{sin82}[/tex] = [tex]\frac{8.4}{sin40}[/tex]

OR

[tex]\frac{sin40}{8.4}[/tex] = [tex]\frac{sin82}{b}[/tex] → B

Answer:

B

Step-by-step explanation:

whihc question is not a good survey question? dont you agree that the financial crisis is essentially over? on average how many hours do you sleep per day?

Answers

Answer:

The correct answer would be option A, Don't you agree that the financial crisis is essentially over.

Step-by-step explanation:

A survey questionnaire is a series of question which is set formally to be asked from a specific sample of a population to gather, analyze and interpret the data obtained from the sample population. So the questions in the survey must be structured, to the point, clear, short and straight. Survey questions are desirably closed ones and the ones which do not need much explanation by the person who is doing the survey. So the first question in this question require an explanation, and also the question is not structured, clear and to the point and thus is not a good survey question. Comparatively, the second question is more structured, clear and to the point and such questions are normally desirable in the surveys.  

Answer:

See image

Step-by-step explanation:

Plato

Is PQR-XYZ? If so, name which similarity postulate or theorem applies.
80°
6
3/80
p5 R
A. Similar - AA
O
O
B. Similar - SSS
O
c. Similar - SAS
D. Cannot be determined​

Answers

Answer:

D. Cannot be determined​

Step-by-step explanation:

there is not enough information for you to answer this question. A picture would be helpful to answer this question, but there isn't one.

The correct option is,

D. Cannot be determined.

What is mean by Triangle?

A triangle is a three sided polygon, which has three vertices and three angles which has the sum 180 degrees.

Since, We are given that two triangle PQR and XYZ are similar.

AA-Similarity postulate: Two angles of one triangle is equal to its corresponding angles of other triangle. Then, the two triangles are similar by AA postulate.

SSS similar: When three sides of two triangles are in the same ratio.

SAS similar: When two sides of two triangle are in the same ratio and one angle between two proportional sides of two triangles is congruent.

But we have not enough information by which we can determine triangle PQR and triangle XYZ is similar.

Hence, option D is true.

Learn more about the triangle visit;

brainly.com/question/1058720

#SPJ7

What is the sum and classification of [tex]\frac{2}{5} + \sqrt{88}[/tex]

A- 9.78083151..., irrational

B - 9.78083151..., rational

C-13.38083151..., irrational

D- 13.38083151..., rational

Answers

Answer:

The answer is def. A.

Answer: OPTION A.

Step-by-step explanation:

By definition:

A number is Rational when it can be written as a simple fraction.

A number is Irrational when it cannot be written as a simple fraction.

Notice that:

[tex]\frac{2}{5}[/tex] is a Rational number.

[tex]\sqrt{88}=2\sqrt{2}[/tex] It is an irrational number.

Therefore, the sum of these numbers is an Irrational number:

[tex]\frac{2}{5} + \sqrt{88}=\frac{2}{5} + 2\sqrt{22}=9.78083151...[/tex]

find the x and y intercepts for the equation 2x-5y=6

Answers

Answer:

X intercepts (3,0) Y (0, -6/5)

Step-by-step explanation:

X intercepts: -6-2x/5=0

                       5(-6-2/5)=0*5

                         -6+2x=0

                          -6+6+2x=0+6

                          2x=6

                           2x/2=6/2

                            x=3

Y intercepts: y=-6-2*0/5

                      y=6-2*0=6

                        6-2*0/5

                         6-0/5

                          6/5

Answer:

X= 5y/2+3

Step-by-step explanation:

First add 5y to both sides

2x-5y+5y=6+5y

2x=5y+6

Step 2

Divide both sides by 2

2x/2=5y+6/2

X=5y/2+3

25 points!!
Can postulates always be proven true?

Answers

Answer:

The basic answer to your question is that we have to start somewhere.

The essence of mathematics (in the sense the Greeks introduced to the  

world) is to take a small set of fundamental "facts," called  

postulates or axioms, and build up from them a full understanding of  

the objects you are dealing with (whether numbers, shapes, or  

something else entirely) using only logical reasoning such that if  

anyone accepts the postulates, then they must agree with you on the  

rest.

It can sometimes be proven.

If cos(x) = 0.5, then what is x?

Answers

Answer:

=60 the second option.

Step-by-step explanation:

Given the trigonometric ratio, we can find the value of the angle by simply finding the inverse of the given ratio.

If for example Cos ∅= a, then ∅=Cos⁻¹a

If Cos (x)= 0.5, then x= Cos⁻¹ 0.5

Cos⁻1 0.5=60°

The angle whose sine is 0.5 is ∅=60°

Answer: second option.

Step-by-step explanation:

You have that:

[tex]cos(x)=0.5[/tex]

Then, to find the value of "x", you need to apply Arccosine ( This is the inverse function of the cosine).

Therefore, applying this in the procedure, you get that the value of "x" is the following:

[tex]cos(x)=0.5\\\\x=Arccos(0.5)\\\\x=60\°[/tex]

You can observe that this matches with the second option.

What is the value of a?

Answers

Check the picture below.

notice, on the first 2 lines, we used the quadrilateral conjecture.

on the lines 3 and 4 we used the inscribed angle theorem.

line 5, if we add those 3 arcs, we end up with 427°, mind you that the surplus is arcYW or "b", since it's added twice by XW and YZ.

line 6, if we subtract a full circle from it, we have the surplus.

line 7, we simply subtract YW from XW, leaving the leftover of arc "a".

Using the data: 2, 2, 3, 3, 3, 4, 5, 6, 6, 19

What is Q1 and Q3

3.5, 6.5

2, 10

3, 6

2, 5

Answers

Answer: Third Option

[tex]Q_1=3[/tex]

[tex]Q_3=6[/tex]

Step-by-step explanation:

Notice that we already have the data sorted from least to greatest.

Now to find Q1 and Q3 we can use the following formulas

For a set of data ordered from least to greatest of the form [tex]X_1, X_2, ..., X_n[/tex]

Where n is the total number of data

[tex]Q_1=X_{\frac{1}{4}(n+1)}[/tex]

In this case [tex]n=10[/tex]

So:

[tex]Q_1=X_{\frac{1}{4}(10+1)}[/tex]

[tex]Q_1=X_{2.75}[/tex]

Round the nearest whole and get:

[tex]Q_1=X_{3}[/tex]

[tex]Q_1=3[/tex]

For [tex]Q_3[/tex] we have:

[tex]Q_3=X_{\frac{3}{4}(n+1)}[/tex]

[tex]Q_3=X_{\frac{3}{4}(10+1)}[/tex]

[tex]Q_3=X_{8.25}[/tex]

Round the nearest whole and get:

[tex]Q_3=X_{8}[/tex]

[tex]Q_3=6[/tex]

Q1 = 3
Q3 = 6

Hope this helps!

Problem solving fractions please help!

Answers

Problem #1= 7 5/12- 6 1/12= 1 4/12 which is simplified to 1 1/3

The answer to the first problem is 1 1/3 feet.

Problem #2= 28(3/4)= 84/4= 21

The answer to the second problem is 21 points.

Hope this helps!

The answer to the first problem is 1 1/3 feet and the second problem is 21 points. The explanation is shown in the picture.

Please help with this one question ASAP. (show steps if you can)

Answers

For this case we have that the area of the figure is given by the area of a rectangle plus the area of a square. By definition, the area of a rectangle is given by:

[tex]A = a * b[/tex]

According to the figure we have:

[tex]a = 9 \sqrt {2}\\b = 8 \sqrt {2} -2 \sqrt {2} = 6 \sqrt {2}[/tex]

So, the area of the rectangle is:

[tex]A = 9 \sqrt {2} * 6 \sqrt {2} = 54 (\sqrt {2}) ^ 2 = 54 * 2 = 108[/tex]

On the other hand, the area of a square is given by:

[tex]A = l ^ 2[/tex]

Where:

l: it is the side of the square

According to the figure we have:

[tex]l = 2 \sqrt {2}[/tex]

So:

[tex]A = (2 \sqrt {2}) ^ 2 = 4 * 2 = 8[/tex]

Finally, the area of the figure is:

[tex]A_ {t} = 108 + 8 = 116[/tex]

Answer:

116

Other Questions
which graph represents a function with direct variatio? please help!!! Which ordered triple represents all of the solutions to the system of equations shown below?2x - 2y - z = 6-x + y + 3z = -33x - 3y + 2z = 9a(-x, x + 2, 0)b(x, x - 3, 0)c(x + 2, x, 0)d(0, y, y + 4)What is the solution to the system of equations shown below?2x - y + z = 44x - 2y + 2z = 8-x + 3y - z = 5a (5, 4, -2)b (0, -5, -1)c No Solutiond Infinite Solutions The point slope form of the equation of the line that passes through (-4, -3) and (12, 1) is y-1=1/4(x-12) what is the standard formula equation for this line? To pass science, a student must earn at least a grade of 70. How many students failed this science class? How did the beliefs held by Mohandas Gandhi and H0 Chi Minh compare? A.Both believed armed conflict was the only way to establish a new government. B.Both believed their country should be divided up along religious lines. C.Both believed communism should be spread to neighboring countries. D.Both believed colonial powers were exploiting the citizens of their country. If h(x) is the inverse of f(x), what is the value of h(f(x))?0 1o f(x) While driving in heavy traffic, Nathan narrowly misses hitting another car. After the event, his heart rate and blood pressure increase. According to Selye, Nathan is MOST likely in which phase of the general adaptation syndrome? (A) resistance (B) exhaustion(C) alarm reaction(D) primary appraisal Explain why the symptoms of celiac disease are consistent with the effects of celiac disease on the small intestine. A(n)_______ (rational /irrational) number answer is desired. Point W is located on QR so that QW/QR = 3/4. What are the coordinates of point W? Which of the following groups are most likely to grow as a percentage of the U.S. population Asian Americans Whites African Americans All of the above The acceleration of a baseball pitcher's hand as he delivers a pitch is extreme. For a professional player, this acceleration phase lasts only 50 ms, during which the ball's speed increases from 0 to about 90 mph, or 40 m/s.What is the force of the pitcher's hand on the 0.145 kg ball during this acceleration phase? Which shows one way to determine the factors of 4x3 + x2 8x 2 by grouping? Target ROI is 19% Invested Capital is $527956 Full Cost per unit $1446 Expected sales volume is 748 units. If the company prices each unit to earn the target ROI, what amount of profit would be added to the cost of each unit? Round your answer to the nearest whole dollar . Don't use dollar signs or commas when entering your answer. A general environmental analysis can be expected to produce all of the following EXCEPTa. objective answers.b. recognition of environmental trends.c. identification of organizational opportunities.d. identification of organizational threats. A photon has a momentum of 5.55 x 10-27 kg-m/s. (a) What is the photon's wavelength? nm (b) To what part of the electromagnetic spectrum does this wavelength correspond? the tolerance is +/-2% Two friends visited Washington D.C. for the weekend. Person 1 rode the subway one stop, 5 times at the peak fare price and four times at the off peak fare price for a total cost of $17.40. Person 2 rode with Person 1, 2 times at the peak fare price and three times at the off peak fair price for a total cost of $9.20. How much was the peak fare price? Which of the following is something you need to keep an eye out fornear packed intersections?A. Farm equipmentB. Animals crossingC. Increased police presenceD. Pedestrians ignoring DON'T WALK signs Find the coordinates when the parallel lines AB and PQ are reflected over the x-axis. The points are A(1, 1), B(1,4), andP(3, 1). Q(3, 4). What is the conjugated expression?